LSAT and Law School Admissions Forum

Get expert LSAT preparation and law school admissions advice from PowerScore Test Preparation.

User avatar
 Dave Killoran
PowerScore Staff
  • PowerScore Staff
  • Posts: 5852
  • Joined: Mar 25, 2011
|
#41140
Complete Question Explanation
(The complete setup for this game can be found here: lsat/viewtopic.php?t=15614)

The correct answer choice is (B)

If O joined the firm in 1965 and M joined the firm in 1967, then it must be true that I joined the firm in 1968 and that G joined the firm in 1966:
J91_Game_#3_#17_diagram 1.png
However, it is still not possible to determine the exact years in which H, N, K, and J each joined the firm. Therefore, if O joined the firm in 1965 and M joined the firm in 1967, one can determine the years in which exactly two of the other partners joined the firm. It follows that answer choice (B) is correct.
You do not have the required permissions to view the files attached to this post.
 salgado145
  • Posts: 9
  • Joined: Dec 19, 2023
|
#104571
Hi
I need help understanding why is G in 6 since I don't see a chain directly with M. I understand I is in place 8 because would be the only letter left over assuming that this could be a possibility

__K/J_. K/J__H/N___H/N____0___. __G__. ___M__. __I__
61. 62. 63. 64. 65. 66. 67. 68

. Thank you
 Robert Carroll
PowerScore Staff
  • PowerScore Staff
  • Posts: 1787
  • Joined: Dec 06, 2013
|
#104668
salgado145,

Consider what's before G. N and J are, via the third rule. The first two rules put one thing before N and one before J. So G has four things before it, and thus can't be in the first four spots. As you point out, G can't be last either. Since, because of the local condition, there is only one place left, G must be there.

Robert Carroll
 kqw24
  • Posts: 1
  • Joined: Mar 26, 2024
|
#105799
Hello,

If we are told the years in which O and M joined, and it must be true that I and G joined at certain times, why is the answer B for 2 and not D for 4? Am I interpreting the question stem incorrectly?
 Robert Carroll
PowerScore Staff
  • PowerScore Staff
  • Posts: 1787
  • Joined: Dec 06, 2013
|
#105803
kqw24,

The question stem asks about "other" partners. O and M shouldn't be included in the count.

Robert Carroll

Get the most out of your LSAT Prep Plus subscription.

Analyze and track your performance with our Testing and Analytics Package.